آيا شما مايل به راه افتادن تاپيك ماراتن نامساوي مي باشي


  • مجموع رای دهندگان
    339

AHZolfaghari

Well-Known Member
ارسال ها
935
لایک ها
1,654
امتیاز
93
پاسخ : ماراتن نامساوي

کسی راه حل این سوال آخر رو اگه بلده بذاره و گرنه خودتون بذارید راه حلو [mention=19860]reza 73[/mention]
 

Dadgarnia

New Member
ارسال ها
1,350
لایک ها
1,127
امتیاز
0
پاسخ : ماراتن نامساوي

سوال بعد : برای اعداد حقیقی بزرگتر از یک ثابت کنید(همه r ها حقیقی و بزرگتر از یک هستند):


حالت کلی نامساوی بالا اینه که با همون شرایط بالا و
داریم:

تعریف می کنیم
می دونیم که
پس
در بازه ی
محدبه حالا کافیه روی
ینسن بزنیم:
با قرار دادن
حکم نتیجه میشه.

---- دو نوشته به هم متصل شده است ----

سوال بعد:
برای اعداد مثبت
که
نشان دهید:

 
آخرین ویرایش توسط مدیر

حمید آنالیز

Well-Known Member
ارسال ها
1,351
لایک ها
1,322
امتیاز
113
پاسخ : ماراتن نامساوي

دوستان لطفا یه نفر این سوالو حل کنه! سوال ساده ایه!
یه راهی دارم که بدون حلیات!
دو تا حالت داریم یا اون اعداد بزرگتر از یکن که بدیهیه!
اگه کوچکتر از یک باشن طرف راست همیشه کوچکتر از یکه ولی طرف چپ بزرگتر از راست است! خوب فک کنم حالت دومو بیشتر حل کنیم بهتره چون حالات خاصی داره به نظرم با جایگشتی حل میشه مگه نه با شرط این که بدترین حالت و بگیریم و اثبات کنیم دسته به نظرتون راهم؟!

---- دو نوشته به هم متصل شده است ----

دوستان لطفا یه نفر این سوالو حل کنه! سوال ساده ایه!
یه راهی دارم که بدون حلیات!
دو تا حالت داریم یا اون اعداد بزرگتر از یکن که بدیهیه!
اگه کوچکتر از یک باشن طرف راست همیشه کوچکتر از یکه ولی طرف چپ بزرگتر از راست است! خوب فک کنم حالت دومو بیشتر حل کنیم بهتره چون حالات خاصی داره به نظرم با جایگشتی حل میشه مگه نه با شرط این که بدترین حالت و بگیریم و اثبات کنیم دسته به نظرتون راهم؟!
 

Dadgarnia

New Member
ارسال ها
1,350
لایک ها
1,127
امتیاز
0
پاسخ : ماراتن نامساوي

یه راهی دارم که بدون حلیات!
دو تا حالت داریم یا اون اعداد بزرگتر از یکن که بدیهیه!
اگه کوچکتر از یک باشن طرف راست همیشه کوچکتر از یکه ولی طرف چپ بزرگتر از راست است! خوب فک کنم حالت دومو بیشتر حل کنیم بهتره چون حالات خاصی داره به نظرم با جایگشتی حل میشه مگه نه با شرط این که بدترین حالت و بگیریم و اثبات کنیم دسته به نظرتون راهم؟!

---- دو نوشته به هم متصل شده است ----

یه راهی دارم که بدون حلیات!
دو تا حالت داریم یا اون اعداد بزرگتر از یکن که بدیهیه!
اگه کوچکتر از یک باشن طرف راست همیشه کوچکتر از یکه ولی طرف چپ بزرگتر از راست است! خوب فک کنم حالت دومو بیشتر حل کنیم بهتره چون حالات خاصی داره به نظرم با جایگشتی حل میشه مگه نه با شرط این که بدترین حالت و بگیریم و اثبات کنیم دسته به نظرتون راهم؟!
اينجوري كه شما حالت بندي كردين فقط دو حالت براي اون اعداد وجود نداره بيشتر از ده حالت وجود داره! تازه توي حالت دوم واقعا به نظرتون اگه يه همچين چيزي رو توي مرحله دو بنويسيد چند نمره بهتون ميدن؟ كلا نامساوي ها اينجوري حل نميشن مخصوصا كه الان نه تا متغير داريم!
 

حمید آنالیز

Well-Known Member
ارسال ها
1,351
لایک ها
1,322
امتیاز
113
پاسخ : ماراتن نامساوي

اينجوري كه شما حالت بندي كردين فقط دو حالت براي اون اعداد وجود نداره بيشتر از ده حالت وجود داره! تازه توي حالت دوم واقعا به نظرتون اگه يه همچين چيزي رو توي مرحله دو بنويسيد چند نمره بهتون ميدن؟ كلا نامساوي ها اينجوري حل نميشن مخصوصا كه الان نه تا متغير داريم!
میخاستم شمارو حرص بدم میدونم اینا اینجوری حل نمیشم ولی نگاهی متفاوت بهشون داشتم و این سوال امکان خیلی کم داره که روز اول طرح بشه و شاید به عنوان سوال 5 میتونه!خوب ببخشین فقط خواستم یه جور دیگه فک کنم که به نتیجه ای میرسم یا نه!:109::205:فک کنم با ارائه حالت تساوی یه دویی بگیرم!
 

Dadgarnia

New Member
ارسال ها
1,350
لایک ها
1,127
امتیاز
0

m-saghaei

New Member
ارسال ها
338
لایک ها
258
امتیاز
0
پاسخ : ماراتن نامساوي

سول بعد:
براي اعداد مثبت
نشان دهيد:

میایم یه تغییر متغیر انجام میدیم:


مسئله میشه اینکه بگیم سیگما رادیکال a^2 به فرجه 3 بزرگتر مساویه 2تا رادیکال سیگما ab به فرجه 3 . (ببخشید اینجا رو لاتکس ننوشت! نمیدونم چرا !! )

میایم یه هولدر چهارتایی میزنیم که یه سیگما a تولید شه و رادیکال ها از بین بره:


پس حالا باید بگیم:
یعنی


که اینم حسابی هندسیه چون جمع دوتا پرانتز سمت راستیا رو داریم و حسابی هندسی بزنیم حکم اثبات میشه.

به سوال گذاشتن ادامه بدید! :4:
 
آخرین ویرایش توسط مدیر

AHZolfaghari

Well-Known Member
ارسال ها
935
لایک ها
1,654
امتیاز
93
پاسخ : ماراتن نامساوي

سوال بعد:
براي اعداد حقيقي
با شرط
نشان دهيد:

طبیعتا دو حالت داریم . x مثبت و y منفی باشه یا اینکه هر مثبت باشند .
در حالت اول که بوضوح طرف چپ نامساوی مثبته . اما در طرف راست xy منفی است . قرینه x+y هم که طبق فرض منفیه پس اون پرانتز منفیه .
هم مثبته . چون x+y مثبته . اون ورش هم طبق حسابی هندسی بزرگ تر مساوی قدر مطلق xy هست . پس در مجموع طرف راست منفی شد که بدیهی است .
حالا حالتی که هر دو مثبت باشند .
S رو برابر جمع دوعدد و P رو برابر حاصلضرب دو عدد قرار میدیم . ( با استدلالی مشابه میشه نتیجه گرفت که نامساوی رو باید فقط برای حالتی که
هست اثبات کرد در غیر این صورت طرف راست نامثبت می شود )
حالا باید ثابت کنیم :





طبق حسابی هندسی :





پس حالا باید ثابت کنیم :






طبق حسابی هندسی داریم :





پس کافیه بگیم :





یعنی :



در واقع :



که بدیهی است و واضح !

سوال بعد :
برای سه عدد مثبت a,b,c ثابت کنید :



 

m-saghaei

New Member
ارسال ها
338
لایک ها
258
امتیاز
0
پاسخ : ماراتن نامساوي

سوال بعد :
برای سه عدد مثبت a,b,c ثابت کنید :


اینجوری درسته؟

میایم رو سه تای اولی کوشی میزنیم:


یعنی باید بگیم:


حالا اگه بگیریم
باید بگیم
و میدونیم که
چون


اگه
باشه که حکم بدیهیه پس فرض میکنیم که


حالا میایم حکم جدید رو به توان دو میرسونیم
یعنی
یعنی باید بگیم


میایم حکم رو تجزیه میکنیم.یعنی باید بگیم
که این هم چون
بدیهیه !!
 

AHZolfaghari

Well-Known Member
ارسال ها
935
لایک ها
1,654
امتیاز
93
پاسخ : ماراتن نامساوي

اینجوری درسته؟

میایم رو سه تای اولی کوشی میزنیم:


یعنی باید بگیم:


حالا اگه بگیریم
باید بگیم
و میدونیم که
چون


اگه
باشه که حکم بدیهیه پس فرض میکنیم که


حالا میایم حکم جدید رو به توان دو میرسونیم
یعنی
یعنی باید بگیم


میایم حکم رو تجزیه میکنیم.یعنی باید بگیم
که این هم چون
بدیهیه !!
بله . راه حل مد نظر من هم همین بود اما تیکه آخرش رو بهتر هم میشد تموم کرد :



به همین راحتی ! بنابر حسابی هندسی !! سوال بعد رو لطف کنید .
 

m-saghaei

New Member
ارسال ها
338
لایک ها
258
امتیاز
0
پاسخ : ماراتن نامساوي

سوال بعد:

داریم
و
ثابت کنید
 
آخرین ویرایش توسط مدیر

AHZolfaghari

Well-Known Member
ارسال ها
935
لایک ها
1,654
امتیاز
93
پاسخ : ماراتن نامساوي

سوال بعد:

داریم
و
ثابت کنید
اول باتوجه به نامساوی جایگشتی می دونیم :


حالا دو طرفو ضربدر xyz میکنیم :



حکم :



بنابر حسابی هندسی :








حالا جمع که بزنیم :




پس





و در نهایت




---- دو نوشته به هم متصل شده است ----

یه راه حل دیگه برای این سوال :


از دو طرف عبارت xy + yz + zx رو کم می کنیم .​



و

طرف چپ به این شکل SOS میشه . طرف راست هم که SOS هست اصلا ! :

پس باید ثابت کنیم :​









پس اگه ثابت کنیم :​
مساله حل میشه .​
که خودش مثبت هست پس باید دومی رو بگیم :​




پس نامساوی اثبات شد و حالت تساوی زمانی است که هر سه برابر باشند .
[MENTION=19852]m-saghaei[/MENTION]​
 
بالا